graph these equations? y = 3x - 5, y = 1/2x + 5

Answers

Answer 1

The solution of the given system of equations is (4, 7).

What is a linear system of equations?

A system of linear equations consists of two or more equations made up of two or more variables such that all equations in the system are considered simultaneously. The solution to a system of linear equations in two variables is any ordered pair that satisfies each equation independently.

The given system of equations are y=3x-5 -------(I) and y=1/2 x+5 -------(II)

Substitute equation (I) in (II), we get

3x-5=1/2 x+5

3x-1/2 x=5+5

2.5x=10

x=4

Substitute x=4 in equation (I), we get

y=3(4)-5

y=7

So, solution is (4, 7)

Therefore, the solution of the given system of equations is (4, 7).

To learn more about the linear system of an equations visit:

https://brainly.com/question/27664510.

#SPJ1

Graph These Equations? Y = 3x - 5, Y = 1/2x + 5

Related Questions

show that the improper integral g(x0dx is divergent but the average value of g on the interval 4,infinoty is finite

Answers

To show that the improper integral g(x)dx is divergent, but the average value of g on the interval 4, infinity is finite, we can consider a function g(x) = 1/x, which is mostly used as an example for a divergent integral.

The improper integral of g(x) from 4 to infinity is given by:

[tex]\int\limits {g(x)} \, dx = \int\limits {(1/x)} \, dx[/tex]  from 4 to infinity

= lim (upper limit -> infinity) [ln|x|] from 4 to x

= [ln|x|] from 4 to infinity

= ln(infinity) - ln(4) = infinity - ln(4) = infinity

As the limit of the integral is infinity, the integral is divergent.

However, the average value of g on the interval [4, ∞) is given by:

average value = (1/∞ - 4) * ∫(1/x)dx from 4 to infinity

= (1/∞ - 4) * (infinity - ln(4))

= -4 + ln(4)

= ln(4) - 4

which is a finite value when the integral is divergent

To learn more about improper integral:

https://brainly.com/question/21023287

#SPJ4

What is the effect on the graph of f(x) = x² when it is transformed to
• h(x) = 5x2 + 10?

Answers

The transformation applied is the one in option D, a vertical dilation of scale factor of 5, and a shift of 10 units up.

What is the effect of the transformation?

Here we start with the parent quadratic function:

f(x) = x²

And we have the transformed function:

h(x) = 5x² + 10

We can write this as:

A vertical dilation of a scale factor 5, which will give:

h(x) = 5*f(x)

And then a translation of 10 units upwards, which gives:

h(x) = 5*f(x) + 10

Replacing the function f(x) we will get:

h(x) = 5*x² + 10

Then the correct option is D.

Learn more about transformations at:

https://brainly.com/question/4289712

#SPJ1

Standardized tests for certain subjects, given to high school students, are scored on a scale of 1 to 5. Let A represent the score on a randomly selected exam for subject A and let B represent the score on a randomly selected exam for subject B. The distributions of scores for each subject’s standardized tests are displayed in the table and the histograms.

Answers

The probability of a score lower than three is given as follows:

0.38.

How to obtain the probabilities?

A probability is obtained as the division of the number of desired outcomes by the number of total outcomes.

As the table gives the probability distribution, we must just take the probabilities of the desired events from the table.

The probability of a score lower than three is given as follows:

P(X < 3) = P(X = 1) + P(X = 2).

Taking the values from the table, the probability is of:

P(X < 3) = 0.18 + 0.20

P(X < 3) = 0.38.

Missing Information

The problem is given by the image presented at the end of the answer.

More can be learned about probabilities at https://brainly.com/question/27899440

#SPJ1

What is 2.4 divided by -0.06?
(With work pls)

Answers

Answer:

To find the result of 2.4 divided by -0.06, we need to use the following steps:

Step 1: Change the division problem into a multiplication problem by flipping the divisor and multiplying by its reciprocal.

-0.06 ÷ 2.4 = -0.06 x 1/(2.4)

Step 2: To find the reciprocal of 2.4, we need to divide 1 by 2.4

1/(2.4) = 0.416666666666667

Step 3: Multiply the original dividend (2.4) by the reciprocal of the divisor (0.416666666666667)

2.4 x 0.416666666666667 = -0.1

So, 2.4 divided by -0.06 is equal to -0.1

It's worth noting that -0.1 is a negative number which makes sense since the divisor is negative, hence the result is the opposite of the result when dividing by a positive number.

What is the distance between the points (-7.5, -11) and (2, -11)?

Answers

Answer:

9.5

Step-by-step explanation:

Distance (d) = [tex]\sqrt{(2-(-7.5)^2)+(-11-(-11)^2)}\\ \sqrt{(9.5)^2+(0)^2}\\ \sqrt{90.25} \\9.5[/tex]

NO LINKS!!!

55, Write an equation satisfying the given conditions.

Answers

Part (a)

The two limit statements tell us that this an exponential decay function.

The curve goes up forever when heading to the left (negative infinity) as indicated by the notation [tex]\displaystyle \lim_{\text{x}\to-\infty}f(x) = \infty[/tex]

At the same time, the curve slowly approaches the horizontal asymptote y = -2, when moving to the right, because of this notation [tex]\displaystyle \lim_{\text{x}\to\infty}f(x) = -2[/tex]

An exponential decay function like [tex]\text{y} = (0.5)^{\text{x}}[/tex] has a horizontal asymptote of y = 0, aka the x axis. The y value approaches 0 but never gets there. Each output is positive.

Shift everything down 2 units to arrive at [tex]\text{y} = (0.5)^{\text{x}}-2[/tex] and this will move the horizontal asymptote down the same amount.

There's nothing really special about the 0.5; you can replace it with any value in the interval 0 < b < 1.

---------

Answer: [tex]\text{f(x)} = (0.5)^{\text{x}}-2[/tex]

====================================================

Part (b)

I'll use this template

[tex]\text{y} = ab^{\text{x}}+c[/tex]

Plugging in x = 0 leads to y = a+c which is the y intercept. Set this equal to the stated y intercept 7 and we get a+c = 7.

We want the [tex]ab^{\text{x}}[/tex] portion to approach zero, which leads to c = 4 so we approach the stated horizontal asymptote.

So,

a+c = 7

a+4 = 7

a = 7-4

a = 3

We go from this

[tex]\text{y} = ab^{\text{x}}+c[/tex]

to this

[tex]\text{y} = 3b^{\text{x}}+4[/tex]

The value of b doesn't matter.

I'll go for b = 0.7 so we get to [tex]\text{f(x)} = 3(0.7)^{\text{x}}+4[/tex]

---------

Answer:  [tex]\text{g(x)} = 3(0.7)^{\text{x}}+4[/tex]

====================================================

Part (c)

The parent function [tex]\text{y} = \log(\text{x}})[/tex] has a domain of [tex](0, \infty)[/tex]. In other words it is the interval [tex]0 < \text{x} < \infty[/tex]

If we replaced each input x with x-5, then we shift the xy axis 5 units to the left. It gives the illusion the log curve moves 5 units to the right.

The vertical asymptote also moves 5 units to the right. We go from a domain of [tex](0, \infty)[/tex] to a domain of [tex](5, \infty)[/tex]

The base of the log doesn't matter.

---------

Answer:  [tex]\text{h(x)} = \log(\text{x}-5)[/tex]

Check out the graphs below. I used GeoGebra, but Desmos is another good option.

(1 point) the following question is from the four steps of statistical inference. if you respond with a summary of all four steps, you will receive no credit on this problem: what is assumed when considering the likelihood of the occurrence of an outcome? hint: this is a question about statistics, not probability.

Answers

When evaluating the likelihood of an outcome, the claim or hypothesis statement is assumed. The required answer is a hypothesis claim or statement.

It is possible to test one's assumptions regarding a population parameter through the statistical analysis known as hypothesis testing. The association between two statistical variables can then be estimated by this hypothesis.

We start by writing the claim mathematically and determining if it is the null or alternative hypothesis. The null hypothesis is the assertion that there is no change or no relationship between the two variables; otherwise, the alternative hypothesis is the assertion that tells that there is a change or relationship.

To know more about the null hypothesis:

https://brainly.com/question/28920252

#SPJ4

part a: the number of transistors per ic in 1972 seems to be about 4,000 (a rough estimate by eye). using this estimate and moore's law, what would you predict the number of transistors per ic to be 20 years later, in 1992? prediction

Answers

Using Moore's Law, which states that the number of transistors on a chip doubles approximately every two years, the estimated number of transistors per IC in 1992 would be 64,000.

The law claims that we can expect the speed and capability of our computers to increase every two years because of this, yet we will pay less for them. In more simple terms, the observation by Gordon Moore in 1965 that the number of transistors in a dense integrated circuit (IC) doubles roughly every two years is known as Moore's law.

This is calculated by doubling the estimate of 4,000 transistors every two years for a total of 8 doublings (16 years).

To know more about Law here

https://brainly.com/question/6590381

#SPJ4

Which inequality is true when x= 4?

Answers

For the value of x = 4, the inequality equation that is true is option C: x/2 < 3.

What is an inequality?

In Algebra, an inequality is a mathematical statement that uses the inequality symbol to illustrate the relationship between two expressions. An inequality symbol has non-equal expressions on both sides. It indicates that the phrase on the left should be bigger or smaller than the expression on the right, or vice versa.

The first inequality equation is - x + 5 < 3

Substitute the value of x = 4 in the equation.

4 + 5 < 3

Now, simplify the equation -

9 < 3

It is known that 9 is not less than 3, but greater than 3.

Therefore, this inequality is false.

The second inequality equation is - 9x > 36

Substitute the value of x = 4 in the equation.

9(4) > 36

Now, simplify the equation -

36 < 36

It is known that 36 is not less than 36, but equal 36.

Therefore, this inequality is false.

The third inequality equation is - x/2 < 3

Substitute the value of x = 4 in the equation.

4/2 < 3

Now, simplify the equation -

2 < 3

It is known that 2 is less than 3.

Therefore, this inequality is true.

The fourth inequality equation is - 18 < x + 8

Substitute the value of x = 4 in the equation.

18 < 4 + 8

Now, simplify the equation -

18 < 12

It is known that 18 is not less than 12, but greater than 12.

Therefore, this inequality is false.

To learn more about inequality from the given link

https://brainly.com/question/11234618

#SPJ1

Which inequality is true when x = 4?

A. x + 5 < 3

B. 9x > 36

C. ×/2 < 3

D. 18 < x + 8

Jose is building a rectangular shaped garden and needs to know how many square feet it will cover. The dimensions of the garden will be 8 feet in length and (3n+2) in width. What is the area of the garden space?

A) 24n+16
B) 11n+10
C) 40n

Thank you!!!​ ​

Answers

Answer:

The equation for the area of a rectangle is length*width. For this problem, it says the length is 8 and the width is (3n+2). All you have to do is multiply 8 by (3n+2).

8(3n+2)=A

24n+16=A

Option A is correct

A cylinder has a height of 7 yards and a diameter of 26 yards. What is its volume? Use ​ ≈ 3.14 and round your answer to the nearest hundredth.

Answers

The volume of the cylinder is 3,714.62 cubic yards.

How to get the volume of the cylinder?

We know that the volume of a cylinder of radius R and height H is given by:

V = pi*R²*H

where pi = 3.14

In this case, we also know that:

H = 7yd

And the diameter is 26 yards, the radius is half of that, then:

R = 26yd/2 = 13yd

Then the volume is:

V = 3.14*(13 yd)²*7yd = 3,714.62 yd³

Learn more about cylinders at:

https://brainly.com/question/9554871

#SPJ1

Assume A,B,C are invertible matrices of appropriate dimension. Solve for X:
AX + B = CX

Answers

The solution of the given equation is X = B(C - A)⁻¹.

To solve for X in the equation AX + B = CX, we can rearrange the equation as follows:

AX - CX = -B

Factor out X on the left side:

(X(A - C)) = -B

To isolate X, we can multiply both sides of the equation by the inverse of (A - C), assuming it exists:

X = -(A - C)⁻¹ × B

X = (C - A)⁻¹ × B

Therefore, the solution for X is:

X = B(C - A)⁻¹

Learn more about Simplification click;

https://brainly.com/question/28261894

#SPJ12

Quadrilateral DEFG is similar to quadrilateral HIJK. Find the measure of side JK.
Round your answer to the nearest tenth if necessary.

Answers

Quadrilateral DEFG is similar to quadrilateral HIJK The measure of side JK is 8.

Since quadrilateral DEFG is similar to quadrilateral HIJK, we know that the ratio of corresponding side lengths is equal. This means that if we know the length of one side of one of the quadrilaterals, we can use that ratio to find the length of the corresponding side in the other quadrilateral.

For example, if we know that side DE is 8 units long, we can use the ratio of side JK to side DE (x/8) to find the length of side JK. We can also use the same ratio of side IJ to side DF which is x/4.

Let's assume that the length of side DE is 8 units. We can use the ratio of side JK to side DE (x/8) to find the length of side JK.

[tex]x/8 = x/4[/tex]

Cross-multiplying and simplifying the equation we get

[tex]4x = 8x[/tex]

[tex]x = 8[/tex]

Therefore, the measure of side JK is 8 units.

Learn more about Quadrilateral

brainly.com/question/29934440

#SPJ4

#7 in need help with this problem please

Answers

You cannot use the Hypotenuse-Leg congruence theorem to prove the similarity of triangles JKM and LKM, as they are not right triangles.

What is the HL congruence theorem?

The Hypotenuse-Leg congruence theorem states that if the hypotenuse and one leg of two right triangles have the same measure then these two triangles are said to be congruent to each other.

The triangles JKM and LKM are not right triangles, as they do not have an angle of 90º, hence the HL congruence theorem cannot be used to prove their similarity.

More can be learned about the Hypotenuse-Leg congruence theorem at https://brainly.com/question/29630133

#SPJ1

I need assistance for this problem below:

Answers

The image of quadratic function f(x) = x² is equal to g(x) = 6 · x².

How to derive and graph the image of a quadratic equation

In this question we see the representation of quadratic equation f(x) = x², from which we need to generate the image of this function, this can be done by a rigid transformation known as vertical dilation:

g(x) = k · f(x), k > 1

Where:

f(x) - Original functiong(x) - Resulting function

If we know that f(x) = x² and k = 6, then the image of function is:

g(x) = 6 · x²

Whose representation on Cartesian plane is shown in the image attached below.

To learn more on dilations: https://brainly.com/question/13176891

#SPJ1

Check image for questions!
(Answers are already there, just gotta figure out where they go)

50 points!

Answers

The equation y is 2x - 3 describes the line with a slope of 2 and a Y-intercept of -3.

Linear equation?

Use m is used to show the slope of a line.

m = 2

c is used to signify the line's y-intercept. c = -3

For the line's equation, the slope-intercept is: y = mx + c

In the equation above, replace c = -3 with m = 2 ,y = 2x - 3

Consequently, the equation of the line with a Y-intercept of -3 and a slope of 2 is as follows: y = 2x - 3.

For the line's equation, the slope-intercept is: y = mx + c

In the equation above, replace c = -3 with m = 2 ,y = 2x - 3

Consequently, the equation of the line with a Y-intercept of -3 and a slope of 2 is as follows: y = 2x - 3.

The complete question is,

The line with a -3 Y-intercept and a 2 slope has what equation?

To learn more about equation of a line refer to:

brainly.com/question/13763238

#SPJ1

For a normal distribution with mean 0 and standard deviation 1, which of the following Python lines outputs the probability P(x<0.325)?
Question 1 options:
a)
import scipy.stats as st
print(st.norm.cdf(0.325, 0, 1))
b)
import pandas as pandas
print(st.norm.pdf(0.325, 0, 1))
c)
import scipy.stats as st
print(st.norm.sf(0.325, 0, 1))
d)
print(normal(0.325, 0, 1))

Answers

Python lines to get the output for the probability P(x<0.325) are -

a) import scipy.stats as

st print(st.norm.cdf(0.325, 0, 1))

Hence, option a is the correct answer.

The Python code -

        import scipy.stats as

        st print(st.norm.cdf(0.325, 0, 1))

This line of code imports the scipy.stats module as "st", and then uses the norm.cdf function to calculate the cumulative distribution function (CDF) of a normal distribution with mean 0 and standard deviation 1, evaluated at x=0.325.

The CDF gives the probability that a random variable from the normal distribution is less than or equal to a certain value, so this line of code is outputting the probability P(x<0.325).

Read more about Probability:

brainly.com/question/24756209

#SPJ4

Sam has 3 1/4 pounds of blueberries. Ben also has some blueberries. Together, they have 9 2/3 pounds of blueberries. Create an equation to represent the number of pounds of blueberries, b, Ben has.

Answers

The required blueberries Ben has 6 5/12 pounds.

What are equation models?

The equation model is defined as the model of the given situation in the form of an equation using variables and constants.

Here,
Let the amount of blueberries Ben has to be x,
According to the question,
Together, they have 9 2/3 pounds of blueberries.
x + 3 1/4 = 9 2/3
x = 9 2/3 - 3 1/4
x = 9 + 2/3 - 3 -1/4
x = 6 + 5/12
x = 6 5/12 pounds

Thus, the required blueberries Ben has 6 5/12 pounds.

Learn more about models here:

https://brainly.com/question/22591166

#SPJ1

Andrew and his brothers want to put their money together to buy a new game system . Costs $168.69 , how much money will each boy need to contribute so they put in equal amount.

Answers

The amount each boy will have to contribute is $56.23

How to calculate the amount that each boy will contribute?

Andrew and his two brother want to contribute money together to buy a new game system

The cost of the game system is $168.69

Each boy want to contribute the same amount

Therefore the amount that each boy will contribute can be calculated as follows

There are 3 boys in total, Andrew and his two brothers

= 168.69/3

= 56.23

Hence each boy will contribute an amount of $56.23

Read more on amount here

https://brainly.com/question/18223891

#SPJ1

a population of bacteria growing exponentially can be modeled as P(t)= P0e kt , where is the time in hours and P0 is the initial population. if the population has a doubling time of 3 hours, calculate the growth constant .

Answers

A population of bacteria growing exponentially can be modeled as P(t)= P0e kt, where is the time in hours and P0 is the initial population. if the population has a doubling time of 3 hours, the growth constant  is k = ln(2) / 3 = 0.231049.

The growth constant (k) can be calculated by taking the natural log of 2 (ln(2)) and dividing it by the doubling time (3 hours). Therefore, the growth constant is 0.231049.  This equation models exponential growth, which is a type of growth where the rate of increase is proportional to the population size. In this case, the population doubles every 3 hours.

This exponential growth is modeled using the equation P(t)= P0e^kt, where P(t) is the population size at a given time, P0 is the initial population size, and k is the growth constant.  By finding the growth constant, we can predict the population size at any given time. The growth constant is found by taking the natural log of 2 (ln(2)) and dividing it by the doubling time (3 hours).

This equation is used to find the amount of growth per unit time and is the same regardless of the population size. This equation can be used to predict the population size at any given time, allowing us to estimate the population size in the future.

To know more about  growth constant reference to the link  brainly.com/question/17120762

#SPJ4

In the equation 10+4y=-4y+2,the variable y represents the same value. Is y = 1, 0, -1, or -2 the solution of this equation explain

Answers

The solution to the equation 10+4y=-4y+2 is -1 because the variable y represents the same value.

What is equation?

An equation is a formula in mathematics that expresses the equality of two expressions by connecting them with the equals sign =. In its most basic form, an equation is a mathematical statement that shows that two mathematical expressions are equal. 3x + 5 = 14, for example, is an equation in which 3x + 5 and 14 are two expressions separated by a 'equal' sign. A mathematical equation that depicts the relationship between two expressions on opposite sides of the sign. It mostly consists of one variable and one equal to symbol. 2x - 4 = 2 is an example.

Here,

10+4y=-4y+2

8y=-8

y=-1

The solution of equation 10+4y=-4y+2 is -1 as the variable y represents the same value.

To know more about equation,

https://brainly.com/question/2228446

#SPJ1

determine the relative order of the metric prefixes of kilo-, micro-, centi-, and milli-. for the same base unit, choose... is less than choose... , which is less than choose... , which is less than choose... .

Answers

The relative order of the metric prefixes Milli- < Centi- < Micro- < Kilo-

The relative order of the metric prefixes of kilo-, micro-, centi-, and milli- is milli-, centi-, micro-, and kilo-. This order is determined by the exponential values of each prefix.

Milli- is equal to 10-3, centi- is equal to 10-2, micro- is equal to 10-6 and kilo- is equal to 103. The exponential values are used to determine the relative order of the prefixes. The lowest exponential value is milli-, making it the smallest metric prefix and the highest exponential value is kilo-, making it the largest metric prefix.

When the exponential values are compared, it is clear that the order of the prefixes is milli- < centi- < micro- < kilo-. This order is used to determine the relative size of the metric prefixes when measuring the same base unit.

For example, if the base unit is meter, then millimeter is the smallest measure and kilometer is the largest measure. The exponential values of the metric prefixes determine the relative order of the base unit.

Learn more about relative order here:

https://brainly.com/question/1623384

#SPJ4

A new apartment building has 33 floors, with 24 apartments on each floor. How many apartments are in the building?

(Partial product)

Red :

Blue :

Green :

Yellow :

Total Product:

Answers

Simple equation when u break it down: 33 x 24=792 apartments

How many apartments are in the building?

A building, or edifice, is an enclosed structure with a roof and walls that is standing in one location more or less permanently, such as a house or factory (although there are also portable buildings). Buildings come in a variety of sizes, shapes, and uses, and they have been modified throughout history for a wide range of reasons, including the availability of building materials, weather, land prices, ground conditions, particular uses, prestige, and aesthetic considerations.

given:

new apartments' building =33 floor

apartments on each =24 floor

33*24=792

792 apartments are in the building

To learn more about building refers to;

https://brainly.com/question/20978526

#SPJ1

PLEASE HELP


A recipe for soup calls for 4 tablespoons of lemon juice and 1/2 cup of olive oil. The given recipe serves 4 ​people, but a cook wants to make a larger batch that serves 120 . ​


a) How many cups of lemon juice will the chef need for the larger​ batch? ​

b) How many pints of olive oil will the chef need for the larger​ batch?

Answers

a) To determine the amount of lemon juice needed for the larger batch, we need to multiply the original amount by the ratio of the larger batch to the original. If the original recipe serves 4 people, and the chef wants to make a batch that serves 120, the ratio is 120/4 = 30.

So the chef will need 30 times the amount of lemon juice that the original recipe calls for.

The original recipe calls for 4 tablespoons of lemon juice, so 4 tablespoons x 30 = 120 tablespoons of lemon juice needed for the larger batch.

Since there are 16 tablespoons in a cup, 120 tablespoons is equivalent to 120/16 = 7.5 cups.

b) To determine the amount of olive oil needed for the larger batch, we will again multiply the original amount by the ratio of the larger batch to the original, which is 30.

The original recipe calls for 1/2 cup of olive oil, so 1/2 cup x 30 = 15 cups of olive oil needed for the larger batch.

Since there are 2 cups in a pint, 15 cups is equivalent to 15/2 = 7.5 pints of olive oil.

So the chef will need 7.5 pints of olive oil for the larger batch.

120 table spoons of lemon juice.

15 cups of olive oil.

if the relation represents a function, find the domain and range. (enter your answers using interval notation. if the relation is not a function, enter none in the domain and range answer blanks.)

Answers

If the relation is represented by the equation y = 2x + 1, the domain would be all real numbers (-infinity, +infinity) and the range would be all real numbers greater than or equal to 1 (1, +infinity).

A relation is a function if for every input (x) there is exactly one output (y). To determine if a relation is a function, we can use the vertical line test, which states that if a vertical line can be drawn through the graph and intersects the relation more than once, then the relation is not a function.

If the relation is a function, we can find the domain and range by analyzing the relation. The domain is the set of all x-values and the range is the set of all y-values. In interval notation, the domain is written as (a, b) and the range is written as (c, d).

So, the domain and range answers depend on the relation which is given and it should be specified.

Read more about Functions:

https://brainly.com/question/22340031

#SPJ4

How many full 2 3/4-in. sheets can be cut from 26 1/8-in. stock?

Answers

The required number of sheets cut from the stock are 9.

What is mixed fraction?

A mixed fraction is one that is represented by both its quotient and remainder. For example, 2 1/3 is a mixed fraction, where 2 is the quotient, 1 is the remainder. An amalgam of a whole number and a legal fraction is a mixed fraction.

According to question:

We have,

We need sheets of length = 2 (3/4) in from 26 (1/8) in stock.

So, 2 (3/4) = 11/4 in

26 (1/8) = 209/8

Then,

Number of sheets =  209/8 / 11/4

Number of sheets = 836/88

Number of sheets = 9.5

So, 9 sheets can be cut.

Thus, required number of sheets are 9.

To know more about mixed fraction visit;

brainly.com/question/29264210

#SPJ1

4 groups of 3 give me answer

Answers

The question 4 groups of 3 can be calculated to be a total of 12 objects across all groups.

How to solve for the total number in the group

The total number of groups are 4 in number

Each of the 4 groups have 3 objects in it.

That is 3 in 4 places

This can be solved by 4 * 3 = 12

Hence we can say that the solution for the questions that says 4 groups of 3 is 12

Read more on grouping here: https://brainly.com/question/5982761

#SPJ1

for each of the following sets of functions either find a function f(x) in their span such that f(x) > 0 for all x

Answers

f(x) = 7x - 1, which is a linear equation with a positive leading coefficient, making f(x) > 0 for all x.

Set 1: {f(x) = x + 1, f(x) = x - 1}

Function f(x) = x^2 + 2 can be found in the span of the two given functions. This can be seen by expanding the equation:

f(x) = x^2 + 2 = (x + 1) + (x - 1) = 2x;

thus, f(x) = 2x + 2, which is a quadratic equation with a positive leading coefficient, making f(x) > 0 for all x.

Set 2: {f(x) = 4x - 2, f(x) = 3x + 1}

Function f(x) = 7x - 1 can be found in the span of the two given functions. This can be seen by expanding the equation:

f(x) = 7x - 1 = (4x - 2) + (3x + 1) = 7x;

thus, f(x) = 7x - 1, which is a linear equation with a positive leading coefficient, making f(x) > 0 for all x.

Learn more about linear equation here:

https://brainly.com/question/11897796

#SPJ4

Complete question:Find a function f(x) in the span of the functions f1(x) = x and f2(x) = -x such that f(x) > 0 for all x.

shola collects N2x salary every month. when he takes away his expenditure of N5000, the salary is less than or equal to N2500. How much does Shila collects every months?

Answers

Answer:

less than or equal to N7500

Step-by-step explanation:

N2x - N5000 =  N2500

N2x = N7500

60⁰
m
Find the area of each triangle round to the nearest tenth
For math thanks !!!!!

Answers

Answer:

  5.2 square units

Step-by-step explanation:

You want the area of a triangle with sides 4 and 3, and the angle between them measuring 60°.

Area

The formula for the area of a triangle is ...

  A = 1/2ab·sin(C)

Application

Here, we have a=4, b=3, C=60°, so the area is ...

  A = 1/2·4·3·sin(60°) = 3√3 ≈ 5.2 . . . . square units

The area of the triangle is about 5.2 square units.

Other Questions
How much change in temperature would the addition of 39,231J of heat have on a 532g sample of copper? lola's grandparents moved to a new neighborhood last year. today, lola can easily remember their new address but messes up their old one. this example illustrates . group of answer choices retroactive interference trace decay proactive interference interference when addressing performance issues as a supervisor, it is best to rely on this to guide your management style: A car weighs 15,000 n and its tires are I flared to a pressure of 190 kPa. How large is the area of the cars tires that are in contact with the road What are complex sentences What are the implications of the differences between the gospel of Mark and the gospel of Matthew? How do you explain them? the need to consider the interests and perspectives of the affected parties in assessing alternative action plans, the other's moral and legal rights, and our duty to treat the other as an end and not as a means, all stand in opposition to the dangerous and myopic philosophy of , which is blind to the rights and aspirations of others. Quadrilateral GFHE and quadrilateral TRQS are similar. Which pair of angles are corresponding congruent angles of these quadrilaterals? the mergers and acquisitions of roche acquiring genentach (pharmaceuticals) and mars acquiring wrigley (candy) are examples of growth strategy, a type of grand strategy. Find the value of a + 16 when a=8. One way that officials in Saudi Arabia have adapted to a lack of water includes Obuilding desalination plants O planting drought resistant crops Oelevating homes to avoid flooding Oconstructing dams on major rivers Who was President when Johnson was first elected to the US House of Representative Can you guys help me with this one? which nursing intervention should a nurse perform when caring for a client who is prescribed opiate therapy for pain? A graphing calculator is recommended.Find the maximum and minimum values of the function. (Round your answer to two decimal places)y=3x-4sin3x, 0x2 Which of the following recommendations supports one of the overarching guidelines of the Dietary Guidelines?A. Consume between 2000 and 2500 kcal/day to prevent undernutritionB. Limit your intake of foods that are high in saturated fatC. Choose foods to achieve optimal health, physical fitness, and well-beingD. Reduce your sodium and potassium intake to no more than 3,000 mg/day One-third of a herd of elephants and three times the square root of the remaining part of the herd were seen on a mountain slope; and in a lake was seen a male elephant along with three female elephants constituting the ultimate(last) remainder. How many were the elephants here? refer to the figure showing the structure of methyl acetate. which statement explains why this compound can form hydrogen bonds with water but not with itself? which of the following groups of laborers found themselves in a position similar to that of the chinese during the gold rush? The table at the right shows the random sample that Jeremy generated from the same population as Morgan's and Maddy's samples. Compare Jeremy's sample to Morgan's and Maddy's.